Sail E0 Webinar

Reasoning Aptitude

STATEMENT AND CONCLUSION MCQs

Statement Conclusions And Inferences

Total Questions : 618 | Page 39 of 62 pages
Question 381. Should the teachers be stopped from beating the students?Arguments:I. Yes, child psychologies say that beating hinders the learning process in a child.II. No, spare the cane and spoil the child.
  1.    if only argument I is strong.
  2.    if only argument II is strong.
  3.    if either I or II is strong.
  4.    if neither I nor II is strong.
  5.    if both I and II are strong.
 Discuss Question
Answer: Option A. -> if only argument I is strong.


Anything which hinders the learning process in a child must be controlled and, if possible, be prohibited. The aim of teacher should be to make a child learn. According to argument I, beating causes hindrance to learning process of a child thus defeating the ultimate goal. If also supports its argument by referring to the findings of psychologists. Hence, I is a strong argument. Argument II states that a child will be spoiled if it is not caned. Caning is an extreme measure, there are also other softer methods to rectify a child. Moreover, the argument is not substantiated by any facts. Hence, II is a weak argument.


Question 382. Should the institute conduct classes in remote villages?Arguments:I. Yes, this will help those students who belong to villages and cannot visit urban ares for studies.II. No, this is not an economically viable proposal, as the number of students who attend such classes cannot contribute to break-even.
  1.    if only argument I is strong.
  2.    if only argument II is strong.
  3.    if either I or II is strong.
  4.    if neither I nor II is strong.
  5.    If both I and II are strong.
 Discuss Question
Answer: Option E. -> If both I and II are strong.


Statement I: The basic purpose of conducting classes is to help students. Hence, statement I is a strong argument as it conveys this idea.Statement II: If the institute conducts classes with the intention of making profits, then this is a valid point to be considered. Hence, statement II is also strong.


Question 383. Should concrete roads be built all over the country instead of bitumen roads?Arguments:I. Yes, as concrete roads lasts 10 times longer than bitumen roads, saves 20% in terms of fuel burnt by vehicles and is cheaper to lay than bitumen roads.II. No, bitumen is 3 times cheaper, saves 15% of fuel burnt by vehicles and is half as costly to lay than the concrete roads. 
  1.    if only argument I is strong.
  2.    if only argument II is strong.
  3.    if either I or II is strong.
  4.    if neither I nor II is strong.
  5.    if both I and II are strong.
 Discuss Question
Answer: Option C. -> if either I or II is strong.


Statement I highlights the advantages of using concrete vis-a-vis bitumen and from if we know that concrete is supreme in every aspect to bitumen and hence it should be used to builds roads. Hence, I is strong.
Statement II gives a similar arguments as in the above case but favouring bitumen. Hence, II is also strong.However, both I and II contradict each other and hence they cannot both be strong at the same time. Hence, either I or II is strong but not both.


Question 384. Should the government stop giving licenses to the new private educational institutions?Arguments:I. Yes, the existing educational institutions themselves are suffering with qualified faculty shortage.II. No, it will hamper the growth.
  1.    if only argument I is strong.
  2.    if only argument II is strong.
  3.    if either I or II is strong.
  4.    if neither I nor II is strong.
  5.    if both I and II are strong.
 Discuss Question
Answer: Option A. -> if only argument I is strong.


It is based on the existing problem and further it will worsen the situation. Hence I is strong. II is simple and it is not giving any reason why it should be encouraged, hence II is not a strong argument.
Only I is strong.


Question 385. Should internet shopping be encouraged?Arguments:I. No, the regular markets will be affected badly.II. Yes, it provides wider range of products, providing a wider range of choice.
  1.    if only argument I is strong.
  2.    if only argument II is strong.
  3.    if either I or II is strong.
  4.    if neither I nor II is strong.
  5.    if both I and II are strong.
 Discuss Question
Answer: Option B. -> if only argument II is strong.


If the new market is going to hamper the regular market then the regular market should over come this by innovative methods, but we cannot discourage the new markets. So, I is not strong. II is based on positive result and desirable. Hence II is strong.
Only II is strong.


Question 386. Should "Brain Gym exercise" be performed as a classroom exercise in schools?Arguments:I. Yes, brain gym exercises helps in overcoming learning related difficulties.II. Yes, the exercise is very simple and substantially improves the functioning of the brain.
  1.    if only argument I is strong.
  2.    if only argument II is strong.
  3.    if either I or II is strong.
  4.    if neither I nor II is strong.
  5.    if both I and II are strong.
 Discuss Question
Answer: Option E. -> if both I and II are strong.


Overcoming learning related difficulties is a desirable positive result. Hence, I is strong.
II is based on a positive result as it is very simple and substantially improve the functioning of the brain, which is desirable.Hence II is strong.Both I and II are strong.


Question 387. Should personal interviews be removed from the recruitment process for government jobs?Arguments:I. Yes, this is where corruption plays a major role and it has become evident that this can never be eliminated.II. No, the integrity of a person can best be judged only through personal interaction.
  1.    if only argument I is strong.
  2.    if only argument II is strong.
  3.    if either I or II is strong.
  4.    if neither I nor II is strong.
  5.    if both I and II are strong.
 Discuss Question
Answer: Option E. -> if both I and II are strong.


Statement I is pessimistic in nature because it says that corruption can never be eliminated, but we cannot check the validity of the statement and we have to take this statement to be true, in which case it is to be admitted that statement I is a strong argument. Statement II is a very strong argument because a person's integrity is a major consideration for giving him or her a job. Both the arguments are strong.


Question 388. Should students pursue higher education?Arguments:I. Yes, they would be able to earn more money.II. No, there is no need as such.
  1.    if only argument I is strong.
  2.    if only argument II is strong.
  3.    if either I or II is strong.
  4.    if neither I nor II is strong.
  5.    if both I and II are strong.
 Discuss Question
Answer: Option A. -> if only argument I is strong.


Earning more money is desirable positive result. Hence I is strong. II is simple, and no substantial reason is given, hence II is not a strong argument. Only I is strong.


Question 389. Statement:Should there be a dress code for the employees in the office?Arguments:I. Yes, why not?II. No,  dress code makes life monotonous.
  1.    if only argument I is strong.
  2.    if only argument II is strong.
  3.    if either I or II is strong.
  4.    if neither I nor II is strong.
  5.    if both I and II are strong.
 Discuss Question
Answer: Option D. -> if neither I nor II is strong.


I is not strong as there is no argument.II is simplistic. II is not strong.Neither I nor II is strong.


Question 390. Statement:Should government improve drainage system in the city?Arguments:I. Yes, the existing drainage system was built to eater to 500 households while the number of households increased to 1000.II. No, there is no improvement in the infrastructure in the city.
  1.    if only argument I is strong.
  2.    if only argument II is strong.
  3.    if either I or II is strong.
  4.    if neither I nor II is strong.
  5.    if both I and II are strong.
 Discuss Question
Answer: Option A. -> if only argument I is strong.


I is strong because it states that the existing drainage system will not be sufficient to cater to demand.
II is irrelevant as it is referring to the infrastructure of the city.Only I is strong.


Latest Videos

Latest Test Papers